UG Element Math Solutions Part 2

Elements of Mathematics Solutions for Class 11th ( Part – 2 ) – Unbox Goodies

Hey guys, are you one of them who searching for “Elements of Mathematics: Foundation by jeevansons publications” Solutions or you can say Element math solution for a long time and didn’t get it from any where yet so your wait is over now because we are here with complete ‘Element Math Solution’ for class 11th and there is no need of worry for the students of other classes because we upload a different sets of element math solution by jeevansons publication soon of their respective classes it’s on way.

Table of Contents

As a student of Class 11th, you might find Mathematics to be a challenging subject. However, with the right guidance and study material, you can ace this subject with ease. The “Element of Mathematics Solutions” is an excellent resource for students who are looking to strengthen their foundation in Mathematics.

How to get All Chapters Solutions of Elements of Math?

In the series of class 11th you got all the exercises of chapters in the form of parts like part 1, part 2, part 3, etc. If you have any query, problem or you want to ask some question related to elements mathematics solutions of class 11th or have some suggestion which you want to share with us you just go through the comment box which is just placed below the post it’s help us for providing you a better service and we replying your questions, queries or suggestion related to element of mathematics solution for class 11th as soon as possible.

Join Telegram Channel All Updates Coming there

We providing you a genuine study material which has been checked and verified by our professionals. Now days, Students are struggling with their exams and they didn’t get the genuine and complete study material, so due to on the continuous demand of students we bring these complete element math solution which you get in free of cost.

If you want get notified for all our posts or study material which we are uploading in future you just subscribe/follow us through below given subscribe/follow box. This is the only site in which you can believe without any thinking. The main and the most important aim of the site is to overcome the pressure of the students for getting accurate study material for the preparation of their exams.

Elements of Math Solutions for other Classes ( Like Class 9th, 10th, 12th ) ?

Our team also working for other classes Mathematics Solutions like for 9th class, 10th class, 11th class, 12th class and also for bachelor degrees so stay in touch with us by subscribe/follow us by this you will get notified on your email when we upload a new post in future.

How to get Solutions of Elements of Mathematics Solutions?

All Solutions of Elements of Mathematics for All classes are coming soon on this website, So join our are Telegram Channel to get Notify when we upload a new post on website and we also provide a giveaways on our Telegram Channel So you can also participate on it by Joining our Telegram channel.

Solutions Start From Here ( Part – 2 )

Question. In which quadrants do the following angles lie ?

  1. 750° (ii) – 870°

Solution. (i) 750° = 2 x 360° + 30°

Now, 360° = 1 complete revolution.

Thus the revolving line, starting from OX, makes two complete

revolutions in the positive direction and further traces out an angle of 30° in the same direction.

Hence the revolving line lies in the first quadrant.

  1. — 870° = – 2 x 360° – 150°

Thus the revolving line, starting from OX, makes two complete

revolutions in the negative direction and moves further through angle of 150° in the same direction. Hence, the revolving line lies in the third quadrant.

Question. Express in radians the following angles:

(i) 45° (ii) 530° (iii) 40° 20°

(iv) 104° 36′ (v) – 37° 30′ (vi) 15° 15’ 15”

Solution. (i)

{{45}^{\circ }}=45\times \frac{\pi }{{180}} radians =\frac{\pi }{4} radians.

\left[ {\because {{1}^{\circ }}=\frac{\pi }{{180}}\text{ }\!\!~\!\!\text{ radians }\!\!~\!\!\text{ }} \right]

(ii)

{{530}^{\circ }}=530\times \frac{\pi }{{180}} radians

=\frac{{53\pi }}{{18}} radians.

(iii)

{{20}^{\text{ }\!\!'\!\!\text{ }}} ={{\left( {\frac{{20}}{{60}}} \right)}^{\circ }}={{\left( {\frac{1}{3}} \right)}^{\circ }} \left[ {\because {{{60}}^{\text{ }\!\!'\!\!\text{ }}}={{1}^{\circ }}} \right] \begin{array}{*{20}{r}} {} & {~\text{ }\!\!~\!\!\text{ }} \\ {{{{40}}^{\circ }}{{{20}}^{\text{ }\!\!'\!\!\text{ }}}} & {~={{{\left( {40\frac{1}{3}} \right)}}^{\circ }}={{{\left( {\frac{{121}}{3}} \right)}}^{\circ }}} \\ {} & \text{ }\!\!~\!\!\text{ } \\ {} & ~ \\ {} & {} \end{array} =\frac{{121}}{3}\times \frac{\pi }{{180}}\text{ }\!\!~\!\!\text{ radians }\!\!~\!\!\text{ =}\frac{{121\pi }}{{540}}\text{ }\!\!~\!\!\text{ radians}

(iv)

{{36}^{\text{ }\!\!'\!\!\text{ }}}={{\left( {\frac{{36}}{{60}}} \right)}^{\circ }}={{\left( {\frac{3}{5}} \right)}^{\circ }} \left[ {\because {{{60}}^{\text{ }\!\!'\!\!\text{ }}}={{1}^{\circ }}} \right] {{104}^{\circ }}{{36}^{\text{ }\!\!'\!\!\text{ }}}={{\left( {104\frac{3}{5}} \right)}^{\circ }}={{\left( {\frac{{523}}{5}} \right)}^{\circ }}=\frac{{523}}{5}\times \frac{\pi }{{180}}\text{ }\!\!~\!\!\text{ radians} \left[ {\because {{1}^{\circ }}=\frac{\pi }{{180}}\text{ }\!\!~\!\!\text{ radians }\!\!~\!\!\text{ }} \right]

=\frac{{523}}{{900}}\pi \text{ }\!\!~\!\!\text{ radians}\text{. }\!\!~\!\!\text{ }

(v)

{{30}^{\text{ }\!\!'\!\!\text{ }}}={{\left( {\frac{{30}}{{60}}} \right)}^{\circ }}={{\left( {\frac{1}{2}} \right)}^{\circ }} \left[ {\because {{{60}}^{\text{ }\!\!'\!\!\text{ }}}={{1}^{\circ }}} \right] \begin{array}{*{20}{c}} {} \\ {-{{{37}}^{\circ }}{{{30}}^{\text{ }\!\!'\!\!\text{ }}}=\text{ }\!\!~\!\!\text{ }-{{{\left( {37\frac{1}{2}} \right)}}^{0}}=\text{ }\!\!~\!\!\text{ }-{{{\left( {\frac{{75}}{2}} \right)}}^{\circ }}} \end{array}

=\frac{{-75}}{2}\times \frac{\pi }{{180}}\text{ }\!\!~\!\!\text{ radians }\!\!~\!\!\text{ }\!\!~\!\!\text{ }

(vi)

{{15}^{{\text{ }\!\!'\!\!\text{ }\!\!'\!\!\text{ }}}}={{\left( {\frac{{15}}{{60}}} \right)}^{\text{ }\!\!'\!\!\text{ }}}=\text{ }\!\!~\!\!\text{ }{{\left( {\frac{1}{4}} \right)}^{\text{ }\!\!'\!\!\text{ }}} \left[ {\because {{{60}}^{{\text{ }\!\!'\!\!\text{ }\!\!'\!\!\text{ }}}}={1}'} \right] \begin{array}{*{20}{r}} {} & ~ \\ {{{{15}}^{\text{ }\!\!'\!\!\text{ }}}{{{15}}^{{\text{ }\!\!'\!\!\text{ }\!\!'\!\!\text{ }}}}} & {=\text{ }\!\!~\!\!\text{ }{{{\left( {15\frac{1}{4}} \right)}}^{\text{ }\!\!'\!\!\text{ }}}={{{\left( {\frac{{61}}{4}} \right)}}^{\text{ }\!\!'\!\!\text{ }}}} \\ {} & ~ \end{array} =\text{ }\!\!~\!\!\text{ }{{\left( {\frac{{61}}{4}\times \frac{1}{{60}}} \right)}^{0}} \left[ {\because {{{60}}^{\text{ }\!\!'\!\!\text{ }}}={{1}^{\circ }}} \right] ={{\left( {\frac{{61}}{{240}}} \right)}^{\circ }} {{15}^{\circ }}{{15}^{\text{ }\!\!'\!\!\text{ }}}{{15}^{{\text{ }\!\!'\!\!\text{ }\!\!'\!\!\text{ }}}}={{\left( {15\frac{{61}}{{240}}} \right)}^{\circ }} ={{\left( {\frac{{3661}}{{240}}} \right)}^{\circ }} \begin{array}{*{20}{r}} {} & ~ \\ {} & {~=\frac{{3661}}{{240}}\times \frac{\pi }{{180}}\text{ }\!\!~\!\!\text{ radians }\!\!~\!\!\text{ }\!\!~\!\!\text{ }\!\!~\!\!\text{ }\!\!~\!\!\text{ }\!\!~\!\!\text{ }\!\!~\!\!\text{ }\!\!~\!\!\text{ }\!\!~\!\!\text{ }\!\!~\!\!\text{ }\!\!~\!\!\text{ }\!\!~\!\!\text{ }\!\!~\!\!\text{ }\!\!~\!\!\text{ }\!\!~\!\!\text{ }\!\!~\!\!\text{ }\!\!~\!\!\text{ }\!\!~\!\!\text{ }\!\!~\!\!\text{ }\!\!~\!\!\text{ }\!\!~\!\!\text{ }\!\!~\!\!\text{ }\!\!~\!\!\text{ }\!\!~\!\!\text{ }\!\!~\!\!\text{ }} \\ {} & {} \end{array}

\left[ {\because {{1}^{\circ }}=\frac{\pi }{{180}}\text{ }\!\!~\!\!\text{ radians }\!\!~\!\!\text{ }} \right]

=\frac{{3661\pi }}{{43200}}\text{ }\!\!~\!\!\text{ radians}\text{. }\!\!~\!\!\text{ }

Question. Find the degree measures corresponding to the following radian measures :

(i) {{\left( {\frac{\pi }{8}} \right)}^{\text{e}}}

(ii) {{\left( {\frac{{7\pi }}{{12}}} \right)}^{e}}

(iii) \text{ }\!\!~\!\!\text{ }{{(-2)}^{\text{e}}}

(iv) {{\left( {\frac{3}{4}} \right)}^{e}}

(v) {{(6)}^{\text{e}}}

Solution. Since \pi radians ={{180}^{\circ }}, therefore

(i) \begin{array}{*{20}{r}} {\left( {\frac{\pi }{8}} \right)\text{ }\!\!~\!\!\text{ radians }\!\!~\!\!\text{ }} & {~={{{\left( {\frac{\pi }{8}\times \frac{{180}}{\pi }} \right)}}^{\circ }}={{{\left( {\frac{{180}}{8}} \right)}}^{\circ }}={{{\left( {\frac{{45}}{2}} \right)}}^{\circ }}={{{\left( {22\frac{1}{2}} \right)}}^{\circ }}} \\ {} & {} \end{array} ~={{22}^{\circ }}{{30}^{\text{ }\!\!'\!\!\text{ }}}

(ii) \left( {\frac{{7\pi }}{{12}}} \right) radians ={{\left( {\frac{{7\pi }}{{12}}\times \frac{{180}}{\pi }} \right)}^{\circ }}={{105}^{\circ }}.

(iii) \left( {-2} \right)\text{ }\!\!~\!\!\text{ radians }\!\!~\!\!\text{ } = -{{\left( {\frac{{2\times 180\times 7}}{{22}}} \right)}^{\circ }} \begin{array}{*{20}{r}} {} & ~ \\ {} & {~=\text{ }\!\!~\!\!\text{ }-{{{\left( {\frac{{1260}}{{11}}} \right)}}^{\circ }}=\text{ }\!\!~\!\!\text{ }-{{{\left( {114\frac{6}{{11}}} \right)}}^{\circ }}} \\ {} & {~=-\left[ {{{{114}}^{\circ }}{{{\left( {\frac{{6\times 60}}{{11}}} \right)}}^{\text{ }\!\!'\!\!\text{ }}}} \right]=-\left[ {{{{114}}^{\circ }}{{{\left( {32\frac{8}{{11}}} \right)}}^{\text{ }\!\!'\!\!\text{ }}}} \right]} \end{array} =-{{114}^{\circ }}{{32}^{\text{ }\!\!'\!\!\text{ }}}{{\left( {\frac{{8\times 60}}{{11}}} \right)}^{{\text{ }\!\!'\!\!\text{ }\!\!'\!\!\text{ }}}} =-{{114}^{\circ }}{{32}^{\text{ }\!\!'\!\!\text{ }}}{{44}^{{\text{ }\!\!'\!\!\text{ }\!\!'\!\!\text{ }}}}

(iv)

\left( {\frac{3}{4}} \right)\text{ }\!\!~\!\!\text{ radians}={{\left( {\frac{3}{4}\times \frac{{180}}{\pi }} \right)}^{\circ }}={{\left( {\frac{{135}}{\pi }} \right)}^{\circ }}={{\left( {\frac{{135\times 7}}{{22}}} \right)}^{0}}={{\left( {\frac{{945}}{{22}}} \right)}^{\circ }}\text{ }\!\!~\!\!\text{ } ={{\left( {42\frac{{21}}{{22}}} \right)}^{\circ }}={{42}^{\circ }}+{{\left( {\frac{{21\times 60}}{{22}}} \right)}^{\text{ }\!\!'\!\!\text{ }}} \begin{array}{*{20}{r}} {} & ~ \\ {} & ~ \\ {} & {~={{{42}}^{\circ }}+{{{57}}^{\text{ }\!\!'\!\!\text{ }}}+{{{\left( {\frac{{3\times 60}}{{11}}} \right)}}^{{\text{ }\!\!'\!\!\text{ }\!\!'\!\!\text{ }}}}} \end{array} ={{42}^{\circ }}{{57}^{\text{ }\!\!'\!\!\text{ }}}{{17}^{{\text{ }\!\!'\!\!\text{ }\!\!'\!\!\text{ }}}}\left( {\text{ }\!\!~\!\!\text{ approx }\!\!~\!\!\text{ }} \right)

(v)

6\text{ }\!\!~\!\!\text{ radians }\!\!~\!\!\text{ }={{\left( {6\times \frac{{180}}{\pi }} \right)}^{\circ }}={{\left( {\frac{{1080\times 7}}{{22}}} \right)}^{\circ }}={{\left( {343\frac{7}{{11}}} \right)}^{\circ }} ={{343}^{\circ }}{{\left( {\frac{{7\times 60}}{{11}}} \right)}^{\text{ }\!\!'\!\!\text{ }}}\text{ }\!\!~\!\!\text{ }\!\!~\!\!\text{ }\!\!~\!\!\text{ } \begin{array}{*{20}{r}} {} & ~ \\ {} & {\text{ }\!\!~\!\!\text{ }\left[ {\because {{1}^{\circ }}={{{60}}^{\text{ }\!\!'\!\!\text{ }}}} \right]} \\ {} & ~ \\ {} & ~ \\ {} & ~ \end{array} ={{343}^{\circ }}{{\left( {38\frac{2}{{11}}} \right)}^{\text{ }\!\!'\!\!\text{ }}}={{343}^{\circ }}{{38}^{\text{ }\!\!'\!\!\text{ }}}{{\left( {\frac{2}{{11}}} \right)}^{\text{ }\!\!'\!\!\text{ }}} ={{343}^{\circ }}{{38}^{\text{ }\!\!'\!\!\text{ }}}{{\left( {\frac{{2\times 60}}{{11}}} \right)}^{{\text{ }\!\!'\!\!\text{ }\!\!'\!\!\text{ }}}}

\left[ {\because {1}'={{{60}}^{{\text{ }\!\!'\!\!\text{ }\!\!'\!\!\text{ }}}}} \right]

={{343}^{\circ }}{{38}^{\text{ }\!\!'\!\!\text{ }}}{{11}^{{\text{ }\!\!'\!\!\text{ }\!\!'\!\!\text{ }}}}\text{ }\!\!~\!\!\text{ (approx}\text{.) }\!\!~\!\!\text{ }

Question.

(i) If l=25\text{ }\!\!~\!\!\text{ cm},r=\frac{1}{2}\text{ }\!\!~\!\!\text{ m}, then find the value of \theta .

(ii) Find the radius of the circle in which a central angle of {{60}^{\circ }} intercepts an arc of 37.4 cm (Use \left. {\pi =22/7} \right).

[NCERT]

(iii) Find the length of an arc of a circle of radius 10 cm which subtends an angle of {{45}^{\circ }} at the centre.

Solution. (i) Here l=25\text{ }\!\!~\!\!\text{ cm} and r=\frac{1}{2}\text{ }\!\!~\!\!\text{ m=50 }\!\!~\!\!\text{ cm} \theta =\frac{l}{r}=\frac{{25}}{{50}}=\frac{1}{2}\text{ }\!\!~\!\!\text{ radians}\text{. }\!\!~\!\!\text{ }

(ii) Let r be the radius of the circle

Here l=37.4\text{ }\!\!~\!\!\text{ cm}.,\text{ }\!\!~\!\!\text{ }~~~~~\theta ={{60}^{\circ }}=60\times \frac{\pi }{{180}} radian =\frac{\pi }{3} radian

Now,

r=\frac{l}{\theta }=\frac{{37\cdot 4}}{{\frac{\pi }{3}}}=\frac{{374\times 3\times 7}}{{220}}=35\cdot 7\text{ }\!\!~\!\!\text{ cm}

(iii)

Here \theta ={{45}^{\circ }},\text{ }\!\!~\!\!\text{ }\!\!~\!\!\text{ }\!\!~\!\!\text{ }\!\!~\!\!\text{ }l=?,\text{ }\!\!~\!\!\text{ }\!\!~\!\!\text{ }\!\!~\!\!\text{ }\!\!~\!\!\text{ }r=10\text{ }\!\!~\!\!\text{ cm}

Now,

\theta =\frac{l}{r}\text{ }\!\!~\!\!\text{ }\!\!~\!\!\text{ }\Rightarrow \text{ }\!\!~\!\!\text{ }\!\!~\!\!\text{ }{{45}^{\circ }}=\frac{l}{{10}} \begin{array}{*{20}{r}} {} & ~ \\ l & {~={{{450}}^{\circ }}\times \frac{\pi }{{180}}=\frac{{5\pi }}{2}\text{ }\!\!~\!\!\text{ cm}\text{.}} \end{array}

Question. The difference between two acute angles of a rt. angled triangle is \frac{\pi }{9} radians. Find the angles in degrees.

Solution. The sum of two acute angles of the \text{rt}. angled triangle ={{90}^{\circ }}. Let one acute angle be {{x}^{\circ }}

Then the other angle is {{(90-x)}^{\circ }}

Now, difference of these two angles =\frac{\pi }{9} radians

={{\left( {\frac{\pi }{9}\times \frac{{180}}{\pi }} \right)}^{\circ }}={{20}^{\circ }} \therefore \text{ }\!\!~\!\!\text{ }{{(90-x)}^{\circ }}-{{x}^{\circ }}={{20}^{\circ }} {{90}^{\circ }}-2x={{20}^{\circ }}\Rightarrow 2x={{70}^{\circ }}\Rightarrow x={{35}^{\circ }} \therefore \text{ }\!\!~\!\!\text{ } Other angle ={{90}^{\circ }}-{{35}^{\circ }}={{55}^{\circ }}.

Hence the two angles are {{35}^{\circ }} and {{55}^{\circ }}.

Question. (i) Express both in degrees and radians, the angles of a triangle, whose angles to each other are in the ratio 1:2:3.

(ii) The angles of a triangle are in A.P., the greatest of them being {{80}^{\circ }}; find all the three angles in radians.

(iii) The angles of a triangle are in A.P. If one of them is {{36}^{\circ }}, find all angles in radians.

Solution.

(i) Let the angles of the triangle be {{x}^{\circ }},2{{x}^{\circ }} and 3{{x}^{\circ }}
[Being in the ratio 1:2:3] \begin{array}{*{20}{r}} {\therefore \text{ }\!\!~\!\!\text{ }x+2x+3x} & {~={{{180}}^{\circ }}} \\ {\Rightarrow \text{ }\!\!~\!\!\text{ }6x} & {~={{{180}}^{\circ }}\Rightarrow \text{ }\!\!~\!\!\text{ }x={{{30}}^{\circ }}} \end{array} \therefore Angles in degree are {{30}^{\circ }},{{60}^{\circ }},{{90}^{\circ }}

  1. e.,
\text{ }\!\!~\!\!\text{ }30\times \frac{\pi }{{180}},\text{ }\!\!~\!\!\text{ }\!\!~\!\!\text{ }60\times \frac{\pi }{{180}},\text{ }\!\!~\!\!\text{ }\!\!~\!\!\text{ }90\times \frac{\pi }{{180}} radians.

i.e., \frac{\pi }{6},\frac{\pi }{3},\frac{\pi }{2} radians.

(ii) Let the angles of triangle be {{(a-d)}^{\circ }},{{a}^{\circ }},{{(a+d)}^{\circ }} \therefore \left( {a-d} \right)+a+a+d={{180}^{\circ }}\text{ }\!\!~\!\!\text{ } [Sum of angles of a triangle ={{180}^{\circ }} ] 3a={{180}^{\circ }}\Rightarrow a={{60}^{\circ }}

Also, greatest of all three angles =a+d={{80}^{\circ }} \therefore \text{ }\!\!~\!\!\text{ } Third angle ={{180}^{\circ }}-\left( {{{{80}}^{\circ }}+{{{60}}^{\circ }}} \right)={{40}^{\circ }}. \left[ \because \right.One angle =a={{60}^{\circ }}] \therefore Angles are {{40}^{\circ }},{{60}^{\circ }},{{80}^{\circ }} i.e., 40\times \frac{\pi }{{180}},60\times \frac{\pi }{{180}},80\times \frac{\pi }{{180}} radians,

  1. e., \frac{{2\pi }}{9},\frac{\pi }{3} and \frac{{4\pi }}{9} radians.

(iii)

Let the angles be

{{(a-d)}^{\circ }},\text{ }\!\!~\!\!\text{ }\!\!~\!\!\text{ }{{a}^{\circ }},\text{ }\!\!~\!\!\text{ }\!\!~\!\!\text{ }{{(a+d)}^{\circ }} \therefore \left( {a-d} \right)+a+\left( {a+d} \right)\text{ }\!\!~\!\!\text{ }={{180}^{\circ }}\text{ }\!\!~\!\!\text{ }\Rightarrow \text{ }\!\!~\!\!\text{ }3a={{180}^{\circ }}\text{ }\!\!~\!\!\text{ }\Rightarrow \text{ }\!\!~\!\!\text{ }a={{60}^{\circ }}

Also one of the angle is {{36}^{\circ }}
[Given] \therefore The third angle is {{180}^{\circ }}-\left( {{{{60}}^{\circ }}+{{{36}}^{\circ }}} \right)={{84}^{\circ }}

Hence the three angles are {{36}^{\circ }},{{60}^{\circ }} and {{84}^{\circ }}

i.e., \frac{\pi }{5},\frac{\pi }{3} and \frac{{7\pi }}{{15}} radians.

[ \because {{1}^{\circ }}=\frac{\pi }{{180}} radians ]

Question. The angles of a quadrilateral are in A.P. and the greatest is double the least. Find the least angle in radians.

Solution. Let the angles of the quadrilateral be

{{(a-3d)}^{\circ }},{{(a-d)}^{\circ }},{{(a+d)}^{\circ }},{{(a+3d)}^{\circ }}

Now the sum of the angles of the quadrilateral ={{360}^{\circ }} \therefore \text{ }\!\!~\!\!\text{ }\left( {a-3d} \right)+\left( {a-d} \right)+\left( {a+d} \right)+\left( {a+3d} \right)={{360}^{\circ }} 4a={{360}^{\circ }}\Rightarrow a={{90}^{\circ }} \therefore The angles are {{(90-3d)}^{\circ }},{{(90-d)}^{\circ }},{{(90+d)}^{\circ }},{{(90+3d)}^{\circ }}

Greatest angle of the quadrilateral ={{(90+3d)}^{\circ }}.

and \text{ }\!\!~\!\!\text{ least }\!\!~\!\!\text{ angle }\!\!~\!\!\text{ =(90-3}d{{)}^{\circ }}

Now, \text{ }\!\!~\!\!\text{ } least angle =\frac{1}{2}\times greatest angle
[Given]

  1. e.,
    {{(90-3d)}^{\circ }}=\frac{1}{2}{{(90+3d)}^{\circ }}\Rightarrow {{180}^{\circ }}-6d={{90}^{\circ }}+3d
\Rightarrow \text{ }\!\!~\!\!\text{ }\!\!~\!\!\text{ }9d={{90}^{\circ }}\Rightarrow d=10. \therefore Least angle ={{(90-3d)}^{\circ }}={{(90-30)}^{\circ }}={{60}^{\circ }}=60\times \frac{\pi }{{180}}=\frac{\pi }{3} radians.

Question. (i) Find the magnitude, in radians and degrees, of the interior angle of a regular pentagon.

(ii) Express in radians as well as in degrees the angle of a regular polygon of 40 sides.

Solution. (i) Number of sides in a regular pentagon = 5

\therefore Number of exterior angles in a regular pentagon = 5

Sum of exterior angles of a regular pentagon ={{360}^{\circ }} \therefore Each exterior angle =\frac{{{{{360}}^{\circ }}}}{5}={{72}^{\circ }}\text{ }\!\!~\!\!\text{ } [See footnote on next page] \therefore Each interior angle ={{180}^{\circ }}-{{72}^{\circ }}={{108}^{\circ }} =\left( {108\times \frac{\pi }{{180}}} \right)\text{ }\!\!~\!\!\text{ radians }\!\!~\!\!\text{ =}\left( {\frac{{3\pi }}{5}} \right)\text{ }\!\!~\!\!\text{ radians }\!\!~\!\!\text{ }

(ii) The sum of 40 exterior angles of a regular polygon ={{360}^{\circ }} \therefore \text{ }\!\!~\!\!\text{ } Each exterior angle =\frac{{{{{360}}^{\circ }}}}{{40}}={{9}^{\circ }} \therefore \text{ }\!\!~\!\!\text{ } Each interior angle ={{180}^{\circ }}-{{9}^{\circ }}={{171}^{\circ }} =\left( {117\times \frac{\pi }{{180}}} \right)\text{ }\!\!~\!\!\text{ radians }\!\!~\!\!\text{ =}\left( {\frac{{19\pi }}{{20}}} \right)\text{ }\!\!~\!\!\text{ radians}\text{. }\!\!~\!\!\text{ }

Question. The number of sides of two regular polygons are 5:4 and the difference between their angles is {{9}^{\circ }}. Find the number of sides in the polygons.

Solution. Let the number of sides of two regular polygons be 5x and 4x.

\therefore \text{ }\!\!~\!\!\text{ } Each exterior angle of first polygon ={{\left( {\frac{{360}}{{5x}}} \right)}^{\circ }}

and each exterior angle of second polygon ={{\left( {\frac{{360}}{{4x}}} \right)}^{\circ }} \therefore \text{ }\!\!~\!\!\text{ } Each interior angle of first polygon ={{180}^{\circ }}-{{\left( {\frac{{360}}{{5x}}} \right)}^{\circ }}

and \text{ }\!\!~\!\!\text{ } each interior angle of second polygon ={{180}^{\circ }}-{{\left( {\frac{{360}}{{4x}}} \right)}^{\circ }}

According to the given condition, we have

\begin{array}{*{20}{r}} {} & {\left[ {{{{180}}^{\circ }}-{{{\left( {\frac{{360}}{{5x}}} \right)}}^{\circ }}} \right]-\left[ {{{{180}}^{\circ }}-{{{\left( {\frac{{360}}{{4x}}} \right)}}^{\circ }}} \right]={{9}^{\circ }}} \\ {} & {~\Rightarrow {{{\left( {\frac{{360}}{{4x}}} \right)}}^{\circ }}-{{{\left( {\frac{{360}}{{5x}}} \right)}}^{\circ }}={{9}^{\circ }}} \end{array}

Add a Comment

Your email address will not be published. Required fields are marked *